Harmonic function squared and mean value

Click For Summary
The discussion revolves around proving that for a harmonic function u in a disk K, the inequality |u(x,y)| ≤ (1/(a-√(x²+y²))) * (M/π)^(1/2) holds for all points (x,y) in K, given that the integral of u squared is finite. Participants express confusion over the application of the mean value property for harmonic functions since u squared is not harmonic. Suggestions include using the mean value theorem for multiple integrals to establish bounds, despite the challenges posed by the non-harmonic nature of u squared. The conversation highlights the need for careful integration over appropriate regions within the disk to derive the desired result. Ultimately, the problem emphasizes the interplay between harmonic functions and integral inequalities.
arestes
Messages
84
Reaction score
4

Homework Statement


Let u be a harmonic function in the open disk K centered at the origin with radius a. and
∫_K[u(x,y)]^2 dxdy = M < ∞. Prove that
|u(x,y)| \le \frac{1}{a-\sqrt{x^2+y^2}}\left( \frac{M}{\pi}\right)^{1/2} for all (x,y) in K.

Homework Equations


Mean value property for harmonic functions.



The Attempt at a Solution


I first thought this was easy and directly applied the mean property for harmonic functions, but of course, the square of a harmonic function is not harmonic (unless it's a constant).

I can see this problems begs for the mean value using a ball with radius a-\sqrt{x^2+y^2} which would be entirely inside the disk K but I can't get around the fact that u squared is not harmonic. Help please
 

Attachments

  • bolas.jpg
    bolas.jpg
    7.2 KB · Views: 407
Last edited:
Physics news on Phys.org
Are you integrating over the whole disk K (i.e. a double integral) or as a line integral around the boundary of K.
 
I integrated over the area of the smaller circle inside with radius r=a-\sqrt{x^2+y^2}. This is exactly what I did (I know it's wrong):

u^2(x,y)=\frac{\int_{B_r(x,y)} u^2(w, z)dwdz}{\pi r^2}\leq \frac{\int_K u^2(w, z)dwdz}{\pi r^2}=\frac{M}{\pi r^2}\\<br /> |u(x,y)|\leq \sqrt{\frac{M}{\pi r^2}}
But of course, this would work if u^2 were harmonic, which is seldom the case.

Any ideas? I was thinking maybe applying the maximum principle and somehow get bounds...
 
Last edited:
Question: A clock's minute hand has length 4 and its hour hand has length 3. What is the distance between the tips at the moment when it is increasing most rapidly?(Putnam Exam Question) Answer: Making assumption that both the hands moves at constant angular velocities, the answer is ## \sqrt{7} .## But don't you think this assumption is somewhat doubtful and wrong?

Similar threads

  • · Replies 9 ·
Replies
9
Views
2K
  • · Replies 27 ·
Replies
27
Views
2K
  • · Replies 4 ·
Replies
4
Views
2K
  • · Replies 3 ·
Replies
3
Views
907
Replies
4
Views
2K
  • · Replies 8 ·
Replies
8
Views
2K
  • · Replies 2 ·
Replies
2
Views
2K
  • · Replies 3 ·
Replies
3
Views
3K
  • · Replies 1 ·
Replies
1
Views
2K
  • · Replies 12 ·
Replies
12
Views
3K